JEE Main Sets Relations And Functions: Quiz!

Approved & Edited by ProProfs Editorial Team
The editorial team at ProProfs Quizzes consists of a select group of subject experts, trivia writers, and quiz masters who have authored over 10,000 quizzes taken by more than 100 million users. This team includes our in-house seasoned quiz moderators and subject matter experts. Our editorial experts, spread across the world, are rigorously trained using our comprehensive guidelines to ensure that you receive the highest quality quizzes.
Learn about Our Editorial Process
| By Tanmay Shankar
T
Tanmay Shankar
Community Contributor
Quizzes Created: 491 | Total Attempts: 1,778,503
Questions: 10 | Attempts: 1,788

SettingsSettingsSettings
JEE Main Sets Relations And Functions: Quiz! - Quiz

.


Questions and Answers
  • 1. 

    If A = {p, q, r, s} and B = {7, 8, 9}, find which of the following is a set of ordered pair, are relations from A to B?

    • A.

      R = {(7, p), (9, p)}

    • B.

      R = {(p, 7), (r, 8), (s, 7)}

    • C.

      R = {(7, s), (8, q), (r, 9)}

    • D.

      R = {(8, s), (9, q)}

    Correct Answer
    B. R = {(p, 7), (r, 8), (s, 7)}
    Explanation
    In part (b), the given relation r is a subset of A × B and hence, it is a relation but in all other options, no other relation is a subset of A × B and hence only option (b) is correct.

    Rate this question:

  • 2. 

    In an international conference of 200 people, there are 53 Indian women and 39 Indian men. Of these Indian people, 7 are doctors and 42 are either men or doctors. There are no foreign doctors. How many women doctors are attending the conference?  

    • A.

      1

    • B.

      2

    • C.

      3

    • D.

      4

    Correct Answer
    C. 3
    Explanation
    Given n (M ∪ D) = 42;
    n (M) + n (D) – n (M ∩ D) = n (M ∪ D); 39 + 7 – n (M ∩ D) = 46
    n (M ∩ D) = 4;
    Amongst the men there are only 4 doctors and as there are no foreign doctors so there are total three Indian women doctors because the total no of doctors is 7.

    Rate this question:

  • 3. 

    In a sports hostel, 21 are on the baseball team, 26 are on the hockey team and 29 are on the football team. 14 play hockey and baseball, 15 play hockey and football, and 8 play all three games. How many total noes of players are there?  

    • A.

      43

    • B.

      37

    • C.

      44

    • D.

      52

    Correct Answer
    A. 43
    Explanation
    n (B) = 21, n (H) = 26, n (F) = 29;
    n (H ∩ B) = 14, n (H ∩ F) = 15, n (F ∩ B) = 12, n (B ∩ H ∩ F) = 8;
    Total no of players = n (B ∪ H ∪ F);
    n (B ∪ H ∪ F) = n (B) + n (H) + n (F) – n (H ∩ B) – n (H ∩ F) – n (F ∩ B) + n (B ∩ H ∩ F);
    n (B ∪ H ∪ F) = 21 +26 +29 – 14 – 15 – 12 + 8 = 76 – 41 + 8;
    n (B ∪ H ∪ F) = 43

    Rate this question:

  • 4. 

    • A.

      { α∶ π/2≤α≤5π/6}

    • B.

      { α∶ π≤α≤3π/2}

    • C.

      {α∶π/2≤α≤3π/2}

    • D.

      Both a & b

    Correct Answer
    D. Both a & b
  • 5. 

    Which of the following is correct about the above functions? 

    • A.

      F is a one-one onto function

    • B.

      Z = r (cotθ + tanθ), so |z| = r

    • C.

      F is a one-one but not onto function

    • D.

      None of these

    Correct Answer
    C. F is a one-one but not onto function
    Explanation
    clearly f is a one-one function but not onto function f will always be a real number and even numbers do not have any pre-image.

    Rate this question:

  • 6. 

    • A.

      S = qr

    • B.

      R = qs

    • C.

      Q = rs

    • D.

      None of these

    Correct Answer
    A. S = qr
    Explanation
    Given that q and r are relatively prime, therefore the given relation: qN ∩ rN = sN can be written as (qr) N = sN which implies that:
    s = qr.

    Rate this question:

  • 7. 

    • A.

      {x : 2 ≤ x < 4 }

    • B.

      {x : 2 < x ≤4 }

    • C.

      {x : 2 ≤ x ≤ 4 }

    • D.

      {x : 2 > x < 4 }

    Correct Answer
    A. {x : 2 ≤ x < 4 }
    Explanation
     

    Rate this question:

  • 8. 

    Correct Answer
    C.
  • 9. 

    Which of the following is the empty set?

    Correct Answer
    B.
    Explanation
    From the question it is clearly evident that option (b) is correct as x ∈ R but in the given condition the x belongs to the imaginary values as x = ± 3i.

    Rate this question:

  • 10. 

    Two finite sets have ‘x’ & ‘y’ elements. The total number of subsets of the first set is 56 more than the total number of the second set. Which of the following are the values of ‘x’ & ‘y’?  

    • A.

      7, 6

    • B.

      5, 1

    • C.

      8, 7

    • D.

      6, 3

    Correct Answer
    D. 6, 3

Quiz Review Timeline +

Our quizzes are rigorously reviewed, monitored and continuously updated by our expert board to maintain accuracy, relevance, and timeliness.

  • Current Version
  • Mar 22, 2023
    Quiz Edited by
    ProProfs Editorial Team
  • Aug 13, 2014
    Quiz Created by
    Tanmay Shankar
Advertisement
×

Wait!
Here's an interesting quiz for you.

We have other quizzes matching your interest.